Basel 問題

問題1. (1990年 東工大後期 第2問) $n$ を $2$ 以上の整数とする.
(1) $n-1$ 次多項式 $P_n(x)$ と $n$ 次多項式 $Q_n(x)$ ですべての実数 $\theta$ に対して $$\begin{aligned} \sin(2n\theta)&=n\sin(2\theta)P_n(\sin^2\theta),\\ \cos(2n\theta)&=Q_n(\sin^2\theta) \end{aligned}$$ を満たすものが存在することを帰納法を用いて示せ.
(2) $k=1,\dots,n-1$ に対して $\alpha_k=\left(\sin\dfrac{k\pi}{2n}\right)^{-2}$ とおくと $$P_n(x)=(1-\alpha_1x)\cdots(1-\alpha_{n-1}x)$$ となることを示せ.
(3) $\displaystyle\sum_{k=1}^{n-1}\alpha_k=\frac{2n^2-2}{3}$ を示せ.
(1) $n$ に関する帰納法により示す. $n=2$ のとき $$\begin{aligned} \sin4\theta &=2\sin2\theta\cos2\theta\\ &=2\sin2\theta(1-2\sin^2\theta) \end{aligned}$$ なので $P_2(x)=1-2x$ とすればよい. 実際, これは $1$ 次式である. $$\begin{aligned} \cos4\theta &=1-2\sin^22\theta\\ &=1-2(2\sin\theta\cos\theta)^2\\ &=1-8\sin^2\theta+8\sin^4\theta \end{aligned}$$ なので $Q_2(x)=1-8x+8x^2$ とすればよい. 実際, これは $2$ 次式である.
$n=k$ $(\geqq2)$ で題意を満たす多項式 $P_k(x)$, $Q_k(x)$ が存在すると仮定する. いま, 便宜上のため $P_k(\sin^2\theta)$ を $P_k$ などと略記する. $$\begin{aligned} \sin2(k+1)\theta &=\sin2k\theta\cos2\theta+\cos2k\theta\sin2\theta\\ &=k\sin2\theta P_k(1-2\sin^2\theta)+Q_k\sin2\theta\\ &=(kP_k(1-2\sin^2\theta)+Q_k)\sin2\theta \end{aligned}$$ なので $$P_{k+1}(x)=\frac{k}{k+1}P_k(x)(1-2x)+\frac{1}{k+1}Q_k(x)$$ とすればよい. 実際, こう定めた $P_{k+1}(x)$ の $k$ 次の係数は $P_k(x)$ の $k-1$ 次の係数に $\dfrac{-2}{k+1}$ $(<0)$ を掛けたものと $Q_k(x)$ の $k$ 次の係数に $\dfrac{1}{k+1}$ $(>0)$ を掛けたものとの和であり, $P_2(x)$ の $1$ 次の係数と $Q_2(x)$ の $2$ 次の係数は符号が異なるので, 帰納的に $P_{k+1}(x)$ の $k$ 次の係数は同符号の $2$ つの項が足されることが従うからそれは $0$ でなく, たしかに $k$ 次式である. $$\begin{aligned} \cos2(k+1)\theta &=\cos2k\theta\cos2\theta-\sin2k\theta\sin2\theta\\ &=Q_k(1-2\sin^2\theta)-k\sin2\theta P_k\sin2\theta\\ &=Q_k(1-2\sin^2\theta)-4k\sin^2\theta\cos^2\theta P_k\\ &=Q_k(1-2\sin^2\theta)-4k\sin^2\theta(1-\sin^2\theta)P_k \end{aligned}$$ なので $$Q_{k+1}(x)=Q_k(x)(1-2x)-4kx(1-x)P_k(x)$$ とすればよい. 実際, 上と同様の議論により $k+1$ 次式である.
以上より示された.
(2) $\theta=\dfrac{k}{2n}\pi$ $(k=1,\dots,n-1)$ とおくと $$\sin k\pi=n\sin\frac{k}{n}\pi \cdot P_n\left(\sin^2\frac{k}{2n}\pi\right)$$ となり, $\sin k\pi=0$, $\sin\dfrac{k}{n}\pi\neq0$ なので $$P_n\left(\sin^2\frac{k}{2n}\pi\right)=P_n\left(\frac{1}{\alpha_k}\right)=0$$ が成り立つから因数定理より $$P_n(x)=c(1-\alpha_1x)\cdots(1-\alpha_{n-1}x)$$ と表され, $P_n(x)$ は $n-1$ 次式であったので $c$ は定数である. ここで $x=0$ の場合を考えると, $P_2(0)=Q_2(0)=1$ であり, $$\begin{cases} P_{k+1}(0)=\dfrac{k}{k+1}P_k(0)+\dfrac{1}{k+1}Q_k(0)\\ Q_{k+1}(0)=Q_k(0) \end{cases}$$ であるから帰納的に $c=P_n(0)=1$ が従い, これが示すべきことであった.
(3) $P_n(x)$, $Q_n(x)$ の $1$ 次の係数を $p_n$, $q_n$ とおくと, (2) より $p_n=-(\alpha_1+\cdots+\alpha_{n-1})$ なので, 示すべきことは $$p_n=\frac{2-2n^2}{3}$$ に他ならない. $$\begin{aligned} P_{n+1}(x)&=\frac{n}{n+1}P_n(x)(1-2x)+\frac{1}{n+1}Q_n(x)\\ Q_{n+1}(x)&=Q_n(x)(1-2x)-4nx(1-x)P_n(x) \end{aligned}$$ より $$\begin{aligned} &1+p_{n+1}x+O(x^2)\\ &=\frac{n}{n+1}(1+p_nx+O(x^2))(1-2x)+\frac{1}{n+1}(1+q_nx+O(x^2))\\ &=1+\dfrac{n(p_n-2)+q_n}{n+1}x+O(x^2)\\ &1+q_{n+1}x+O(x^2)\\ &=(1+q_nx+O(x^2))(1-2x)-4nx(1-x)(1+p_nx+O(x^2))\\ &=1+(q_n-4n-2)x+O(x^2) \end{aligned}$$ 係数を比較して $$\begin{cases} p_{n+1}=\dfrac{n(p_n-2)+q_n}{n+1}\\ q_{n+1}=q_n-4n-2 \end{cases}$$ 2番目の式より $n\geqq3$ で $$q_n=q_2+\sum_{k=2}^{n-1}(-4n-2)=-2n^2$$ となり, $q_2=-2\cdot2^2=-8$ より $n=2$ でも成立している. これを1番目の式に代入して整理すると $$(n+1)\left(p_{n+1}+\frac{2}{3}(n+1)^2-\frac{2}{3}\right)=n\left(p_n+\frac{2}{3}n^2-\frac{2}{3}\right)$$ となるので $$n\left(p_n+\frac{2}{3}n^2-\frac{2}{3}\right)=\cdots=2\left(-2+\frac{2}{3}\cdot2^2-\frac{2}{3}\right)=0$$ より $p_n=\dfrac{2-2^n}{3}$ を得, これが示すべきことであった.

なぜこんなよくわからない計算をさせたのでしょうか? 実はこのような和を考えることは Basel 問題: $$\sum_{n=1}^{\infty} \frac{1}{n^{2}}=\frac{\pi^{2}}{6}$$ に関連しているのです. この等式は 1734 年に Euler が $\sin{x}$ の Maclaurin 展開を用いて証明したものですが, 実は初等的に証明できることが知られており, 以下に紹介するように大学入試の問題として機能するほどのものとなっております.

問題2. (2020年 慶医 第3問) 関数 $f(x)$, $g(x)$ を $f(x)=\dfrac{1}{\sin^2 x}$, $g(x)=\dfrac{1}{\tan^2 x}$ と定める.
(1) 定数 $a$ を $a=\boxed{(あ)}$ と定めると, $0 < x < \dfrac{\pi}{2}$ のとき $$f(x)+f\left(\frac{\pi}{2}-x\right)=af(2x)$$ が成り立つ.
(2) 自然数 $n$ に対して $$S_n=\sum_{k=1}^{2^n-1}f\left(\frac{k\pi}{2^{n+1}}\right), T_n=\sum_{k=1}^{2^n-1}g\left(\frac{k\pi}{2^{n+1}}\right)$$ とおく. このとき $S_1$, $S_2$, $S_3$ の値を求めると $$S_1=\boxed{(い)}, S_2=\boxed{(う)}, S_3=\boxed{(え)}$$ である. また $S_n$ と $S_{n+1}$ の間には $S_{n+1}=\boxed{(お)}$ の関係がある. このことから, $S_n$ を $n$ の式で表すと $S_n=\boxed{(か)}$ となる. また $T_n$ を $n$ の式で表すと $T_n=\boxed{(き)}$ である. したがって, $0<\theta<\dfrac{\pi}{2}$ に対して $\sin\theta<\theta<\tan\theta$ であることに注意すると, $$\lim_{n\to\infty}\sum_{k=1}^{2^n-1}\frac{1}{k^2}=\boxed{(く)}$$ がわかる.
(1) $0 < x < \dfrac{\pi}{2}$ のとき, $$\begin{aligned} &f(x)+f\left(\frac{\pi}{2}-x\right)\\ &=\frac{1}{\sin^2x}+\frac{1}{\sin^2(\frac{\pi}{2}-x)}\\ &=\frac{1}{\sin^2x}+\frac{1}{\cos^2x}\\ &=\frac{1}{\sin^2x\cos^2x}\\ &=4\cdot\frac{1}{(2\sin x\cos x)^2}\\ &=\boxed{4}f(2x) \end{aligned}$$
(2) (1) の結果を利用しながら実際に計算してみる.
  • $S_1=f(\frac{\pi}{4})=\frac{1}{\sin^2(\pi/4)}=\boxed{2}$
  • $S_2=f(\frac{\pi}{8})+f(\frac{3\pi}{8})+f(\frac{2\pi}{8})=4S_1+2=\boxed{10}$
  • $S_3=f(\frac{\pi}{16})+f(\frac{7\pi}{16})+f(\frac{2\pi}{16})+f(\frac{6\pi}{16})+f(\frac{3\pi}{16})+f(\frac{5\pi}{16})+f(\frac{4\pi}{16})=4f(\frac{\pi}{8})+4f(\frac{\pi}{4})+4f(\frac{3\pi}{8})+f(\frac{\pi}{4})=4S_2+2=\boxed{42}$
このカラクリをよく観察すると $$S_{n+1}=\boxed{4S_n+2}$$ が成り立っていることがわかるので $$S_n=\boxed{\frac{2}{3}(4^n-1)}$$ いま $g(x)=f(x)-1$ なので $$T_n=S_n-(2^n-1)=\boxed{\frac{2\cdot4^n}{3}-2^n+\frac{1}{3}}$$ したがって, $0<\theta<\dfrac{\pi}{2}$ に対して $$\sin\theta<\theta<\tan\theta$$ であるから $$\frac{1}{\tan^2\theta} < \frac{1}{\theta^2} < \frac{1}{\sin^2\theta}$$ であり, $\theta=\dfrac{k\pi}{2^{n+1}}\in\left(0,\dfrac{\pi}{2}\right)$ $(k=1,\dots,2^n-1)$ を代入して $k$ について足し上げると $$T_n < \frac{4^{n+1}}{\pi^2}\sum_{k=1}^{2^n-1}\frac{1}{k^2} < S_n$$ より $$\frac{\pi^2}{6}\left(1-\frac{1}{24\cdot2^n}+\frac{1}{18\cdot4^{n+1}}\right) < \sum_{k=1}^{2^n-1}\frac{1}{k^2} < \frac{\pi^2}{6}\left(1-\frac{1}{9\cdot4^{n+1}}\right)$$ ではさみうちの原理から $$\lim_{n\to\infty}\sum_{k=1}^{2^n-1}\frac{1}{k^2}=\boxed{\frac{\pi^2}{6}}$$ である.
問題3. (2018年 気象大学校) 数列 $\{S_n\}$ を $$S_n=\sum_{k=1}^{2^{n-1}}\left\{\sin\left(\dfrac{2k-1}{2^{n+1}}\right)\pi\right\}^{-2}\quad (n=1,2,\dots)$$ で定める. 以下の設問に答えよ.
(1) $0 < x < \dfrac{\pi}{2}$ のとき $$\sin x < x < \tan x$$ が成り立つことを示せ.
(2) $0 < x < \dfrac{\pi}{4}$ とする. $$\dfrac{1}{\sin^2x}+\dfrac{1}{\sin^2\left(\dfrac{\pi}{2}-x\right)}$$ を $\sin2x$ を用いて表せ.
(3) $S_{n+1}$ を $S_n$ を用いて表せ.
(4) $\{S_n\}$ の一般項を, 和の形を用いずに表せ.
(5) 数列 $\{T_n\}$ を $$T_n=\sum_{k=1}^{2^{n-1}}\dfrac{1}{(2k-1)^2}\quad(n=1,2,\dots)$$ で定める. $\displaystyle\lim_{n\to\infty}T_n$ を求めよ.
(1)(2) 省略.
(3) 前の仕組みをもう一度よく見てみると $f(\frac{\pi}{4})=2$ が消えているだけなので $S_{n+1}=\boxed{4S_n}$ がわかる.
(4) $S_n=4^{n-1}S_1=\boxed{2^{2n-1}}$.
(5) $0 < x < \dfrac{\pi}{2}$ で $$\dfrac{1}{\sin^2x}-1 < \dfrac{1}{x^2} < \dfrac{1}{\sin^2x}$$ が成り立ち, $x=\dfrac{2k-1}{2^{n+1}}\pi\in\left(0,\dfrac{\pi}{2}\right)$ $(k=1,\dots,2^{n-1})$ を代入して $k$ について足しあげると $$S_n-2^{n-1} < \dfrac{2^{2n+2}}{\pi^2}T_n < S_n$$ を得るので $$\left(\dfrac{1}{8}-\dfrac{1}{2^{n+3}}\right)\pi^2 < T_n < \dfrac{\pi^2}{8}$$ となり, はさみうちの原理より $$\lim_{n\to\infty}T_n=\boxed{\dfrac{\pi^2}{8}}.$$
注. 実は $T_{\infty}$ が計算できたなら Basel 問題は容易な計算によって解決することができる. $$S=\dfrac{1}{1^2}+\dfrac{1}{2^2}+\frac{1}{3^2}+\cdots$$ とおくと $$\dfrac{1}{4}S=\dfrac{1}{2^2}+\dfrac{1}{4^2}+\frac{1}{6^2}+\cdots$$ であるから辺々を引いて $$\dfrac{3}{4}S=1+\dfrac{1}{3^2}+\cdots=T_{\infty}=\dfrac{\pi^2}{8}$$ となるので $$S=\dfrac{\pi^2}{6}$$ がたしかに得られる.
問題4. (2019年 東海医) $i$ を虚数単位とする.
(1) $(1+i)^7=\boxed{ア}$
(2) $(\sqrt{x}+i)^7$ の虚部は $x$ の $3$ 次多項式 $\boxed{イ}$ である. ただし, $\boxed{イ}$ は降べきの順に整理して答えよ.
(3) $(\cos\theta+i\sin\theta)^7$ が実数のとき, $\theta=\boxed{ウ}$, $\theta=\boxed{エ}$, $\theta=\boxed{オ}$ である. ただし, $0 < \boxed{ウ} < \boxed{エ} < \boxed{オ} < \dfrac{\pi}{2}$ とする.
(4) $a=\tan\boxed{ウ}$, $b=\tan\boxed{エ}$, $c=\tan\boxed{オ}$ とおき, 多項式 $\boxed{イ}$ を因数分解すると $$\boxed{イ}=\boxed{カ}\left(x-\boxed{キ}\right)\left(x-\boxed{ク}\right)\left(x-\boxed{ケ}\right)$$ となる. ただし, $\boxed{キ}$ は $a$ を, $\boxed{ク}$ は $b$ を, $\boxed{ケ}$ は $c$ を用いて表せ.
(5) $n$ が自然数のとき $(\sqrt{x}+i)^{2n+1}$ の虚部は $x$ の $n$ 次多項式になる. この多項式の $n$ 次の係数は $\boxed{コ}$, $(n-1)$ 次の係数は $\boxed{サ}$ である. したがって, $$\dfrac{1}{\tan^2\dfrac{1}{2n+1}\pi}+\dfrac{1}{\tan^2\dfrac{2}{2n+1}\pi}+\cdots+\dfrac{1}{\tan^2\dfrac{n}{2n+1}\pi}=\boxed{シ}$$ (6) $0 < \theta < \dfrac{\pi}{2}$ のとき $\sin\theta < \theta < \tan\theta$ より $\dfrac{1}{\tan^2\theta}<\dfrac{1}{\theta^2}<\dfrac{1}{\sin^2\theta}$ が成り立ち, $$\lim_{n\to\infty}\sum_{k=1}^{n}\dfrac{1}{k^2}=\boxed{ス}$$ を得る.
(1) $(1+i)^7=(\sqrt{2}e^{\frac{i\pi}{4}})^7=8\sqrt{2}e^{\frac{7\pi i}{4}}=\boxed{8(1-i)}$.
(2) 展開する際に $\sqrt{x}$ を偶数個選んだ項が純虚数になることに注意すると虚部は $\boxed{7x^3-35x^2+21x-1}$ とわかる.
(3) $(\cos\theta+i\sin\theta)^7=\cos7\theta+i\sin7\theta$ が実数になることは $\sin7\theta=0$ となることに等しく, $0 < \theta < \dfrac{\pi}{2}$ の範囲で $7\theta=\pi,2\pi,3\pi$ すなわち $$\theta=\boxed{\dfrac{\pi}{7}},\boxed{\dfrac{2}{7}\pi},\boxed{\dfrac{3}{7}\pi}.$$
(4) $x=\dfrac{1}{\tan^2\theta}$ とおくと $$(\cos\theta+i\sin\theta)^7=\sin^7\theta(\sqrt{x}+i)^7$$ となるので $$\operatorname{Im}(\cos\theta+i\sin\theta)^7=7x^3-35x^2+21x-1$$ であり, (3) より左辺は $\theta=a,b,c$ で $0$ になるので因数定理により $$\boxed{7}\left(x-\boxed{\dfrac{1}{a^2}}\right)\left(x-\boxed{\dfrac{1}{b^2}}\right)\left(x-\boxed{\dfrac{1}{c^2}}\right)$$ と因数分解できる.
(5) $n$ 次の項は $i$ を $1$ つ選んで出来るので係数は $${}_{2n+1}\mathrm{C}_{1}=\boxed{2n+1}$$ であり, $n-1$ 次の項は $i$ を $3$ つ選んで出来るので係数は $$i^2\cdot{}_{2n+1}\mathrm{C}_{3}=\boxed{-\dfrac{1}{3}(2n+1)n(2n-1)}$$ である. 解と係数の関係より $$\begin{aligned} \boxed{シ}&=-\dfrac{-\dfrac{1}{3}(2n+1)n(2n-1)}{2n+1}\\ &=\boxed{\dfrac{1}{3}n(2n-1)}. \end{aligned}$$
(6) $0<\theta<\dfrac{\pi}{2}$ で $$\dfrac{1}{\tan^2\theta}<\dfrac{1}{\theta^2}<\dfrac{1}{\tan^2\theta}+1$$ が成り立つから, $\theta=\dfrac{k\pi}{2n+1}$ $(k=1,\dots,n)$ を代入して $k$ について足しあげると $$\dfrac{1}{3}n(2n-1)<\dfrac{(2n+1)^2}{\pi^2}\sum_{k=1}^{n}\dfrac{1}{k^2}<\dfrac{1}{3}n(2n-1)+n$$ $$\dfrac{n(2n-1)}{3(2n+1)^2}\pi^2<\sum_{k=1}^{n}\dfrac{1}{k^2}<\dfrac{n(2n+2)}{3(2n+1)^2}\pi^2$$ はさみうちの原理より $$\lim_{n\to\infty}\sum_{k=1}^{n}\dfrac{1}{k^2}=\boxed{\dfrac{\pi^2}{6}}.$$

なお, 次のようにガチの数Ⅲゴリ押しで導出することもできる.

問題5. (2003年 日本女子大 理 自己推薦) $n$ を正の整数または $0$ とする. 次の (1)-(7) に答えよ.
(1) 次の式を示せ. $$\int_{0}^{\frac{\pi}{2}}\sin^n{x}\,dx=\int_{0}^{\frac{\pi}{2}}\cos^n{x}\,dx$$ (2) $\displaystyle I_n=\int_{0}^{\frac{\pi}{2}}\sin^n{x}\,dx=\int_{0}^{\frac{\pi}{2}}\cos^n{x}\,dx$ とおく. 次の式を示せ. $$I_n=\frac{n-1}{n}I_{n-2}\quad (n\geqq2)$$ (3) 次の式を示せ. $$I_n=\begin{cases}\dfrac{n-1}{n}\cdot\dfrac{n-3}{n-2}\cdots\dfrac{3}{4}\cdot\dfrac{1}{2}\cdot\dfrac{\pi}{2}&(n\text{ が偶数のとき})\\\dfrac{n-1}{n}\cdot\dfrac{n-3}{n-2}\cdots\dfrac{4}{5}\cdot\dfrac{2}{3}&(n\text{ が奇数のとき})\end{cases}$$ (4) $S_{n}=\displaystyle\int_{0}^{\frac{\pi}{2}} x^{2} \cos^{2n}x\,dx$ とおく. 次の式を示せ. $$S_{n-1}-\frac{2 n}{2 n-1} S_{n}=\frac{2}{2 n(2 n-1)} I_{2 n}\quad(n \geqq 1)$$ (5) 次の式を示せ. ただし, $N$ は整数で $N\geqq1$ とする. $$S_{N}=\frac{(2 N-1)(2 N-3) \cdots 5 \cdot 3 \cdot 1}{(2 N)(2 N-2) \cdots 4 \cdot 2} \cdot \frac{\pi}{4}\left(\frac{\pi^{2}}{6}-\sum_{n=1}^{N} \frac{1}{n^{2}}\right)$$ (6) 次の式を示せ. ただし, $N$ は整数で $N\geqq1$ とする. $$S_{N} \leqq \frac{1}{2 N+2} \cdot \frac{(2 N-1)(2 N-3) \cdots 5 \cdot 3 \cdot 1}{(2 N)(2 N-2) \cdots 4 \cdot 2}\left(\frac{\pi}{2}\right)^{3}$$ なお, $0 < x < \dfrac{\pi}{2}$ で $x < \dfrac{\pi}{2}\sin{x}$ であることを用いてよい.
(7) 次の式を示せ. $$\sum_{n=1}^{\infty} \frac{1}{n^{2}}=\frac{\pi^{2}}{6}$$
(1)(2)(3) 省略.
(4) $$\begin{aligned} S_n &= \int_0^{\frac{\pi}{2}}x^2\cos ^{2n-1}x\cos xdx \\ &= \left[x^2\cos ^{2n-1}x\sin x\right] _0^{\frac{\pi}{2}}-\int_0^{\frac{\pi}{2}}\{ 2x\cos ^{2n-1}x+(2n-1)x^2\cos ^{2n-2}x(-\sin x)\}\sin xdx \\ &= \frac{1}{n}\int_0^{\frac{\pi}{2}}x\cdot 2n\cos ^{2n-1}x(-\sin x)dx+(2n-1)\int_0^{\frac{\pi}{2}}x^2\cos ^{2n-2}x\sin ^2xdx \\ &= \frac{1}{n}\int_0^{\frac{\pi}{2}}x(\cos ^{2n}x)'dx+(2n-1)\int_0^{\frac{\pi}{2}}x^2\cos ^{2n-2}x(1-\cos ^2x)dx \\ &= \frac{1}{n}\left[ x\cos ^{2n}x\right] _0^{\frac{\pi}{2}}-\frac{1}{n}\int_0^{\frac{\pi}{2}}\cos ^{2n}xdx+(2n-1)\int_0^{\frac{\pi}{2}}x^2\cos ^{2n-2}xdx-(2n-1)\int_0^{\frac{\pi}{2}}x^2\cos ^{2n}xdx \\ &= -\frac{1}{n}I_{2n}+(2n-1)S_{n-1}-(2n-1)S_n \end{aligned}$$ となるから, $$S_n = \frac{2n-1}{2n}S_{n-1}-\frac{1}{2n^2}I_{2n}$$
(5) $N$ に関する帰納法で示す. $N=1$ のときたしかに成立している. $N=k-1$ で成立すると仮定すると $$\begin{aligned} S_k &= \frac{2k-1}{2k}S_{k-1}-\frac{1}{2k^2}I_{2k} \\ &= \frac{2k-1}{2k}\cdot\frac{(2k-3)!!}{(2k-2)!!}\cdot\frac{\pi}{4}\left(\frac{\pi ^2}{6}-\sum_{i = 1}^{k-1}\frac{1}{i^2}\right)-\frac{1}{2k^2}\cdot\frac{(2k-1)!!}{(2k)!!}\cdot\frac{\pi}{2} \\ &= \frac{(2k-1)!!}{(2k)!!}\cdot\frac{\pi}{4}\left(\frac{\pi ^2}{6}-\sum_{i = 1}^{k-1}\frac{1}{i^2}-\frac{1}{k^2}\right) \\ &= \frac{(2k-1)!!}{(2k)!!}\cdot\frac{\pi}{4}\left(\frac{\pi ^2}{6}-\sum_{i = 1}^k\frac{1}{i^2}\right) \end{aligned}$$ より $N=k$ でも成立する. よって示された.
(6) $0 < x < \dfrac{\pi}{2}$ において $x < \dfrac{\pi}{2}\sin x$ であるから, 定積分の単調性より $$\begin{aligned} S_N &\leqq \frac{\pi ^2}{4}\int_0^{\frac{\pi}{2}}\sin ^2x\cos ^{2N}xdx \\ &= \frac{\pi ^2}{4}(I_{2N}-I_{2N+2}) \\ &= \frac{\pi ^2}{4}\left(\frac{(2N-1)!!}{(2N)!!}\cdot\frac{\pi}{2}-\frac{(2N+1)!!}{(2N+2)!!}\cdot\frac{\pi}{2}\right)\\ &= \frac{1}{2N+2}\cdot\frac{(2N-1)!!}{(2N)!!}\left(\frac{\pi}{2}\right) ^3 \end{aligned}$$
(7) $$0 \leqq \frac{(2N-1)!!}{(2N)!!}\cdot\frac{\pi}{4}\left(\frac{\pi ^2}{6}-\sum_{n = 1}^N\frac{1}{n^2}\right) \leqq \frac{1}{2N+2}\cdot\frac{(2N-1)!!}{(2N)!!}\left(\frac{\pi}{2}\right) ^3$$ $$0 \leqq \frac{\pi ^2}{6}-\sum_{n = 1}^N\frac{1}{n^2} \leqq \frac{1}{2N+2}\cdot\frac{\pi ^2}{2}$$ はさみうちの原理より $$\lim\limits_{N \to \infty}\left(\frac{\pi ^2}{6}-\sum_{n = 1}^N\frac{1}{n^2}\right) = 0$$ $$\sum_{n = 1}^\infty\frac{1}{n^2} = \frac{\pi ^2}{6}$$